Vous êtes sur la page 1sur 32

Property Digests

Continuation Part 3............

German Management and Services, Inc. vs Court of


Appeals

HELD: No. The Doctrine of Self-help is not applicable because at the time
when German Management excluded the farmers, theres no longer an
actual or threatened unlawful physical invasion or usurpation. That actual or
threatened unlawful physical invasion by the farmers have already lapsed 12
years ago when they began occupying the said land. In fact, they were
already peaceably farming the land.
What should have been the remedy by German Management?

177 SCRA 495 Civil Law Property Doctrine of Self-Help


In February 1982, the spouses Manuel and Cynthia Jose contracted
with German Management and Services, Inc. for the latter to develop their
landholdings into a residential subdivision. The spouses also executed a
special power of attorney to that effect.
German Management started the project in February 1983, however,
German Management discovered that the land was being possessed by
Ernest0 Villeza et al who were the farmers tilling the said land at that time.
German Management spoke with Villeza et al but the farmers refused to
vacate the land as the farmers claimed that they have been occupying the
land for twelve years.
Nevertheless, German Management went on to develop the property and
demolished the properties of the farmers without acquiring a court order. In
turn, Villeza et al filed a case of forcible entry against German Management.
In its defense, German Management invoked the Doctrine of Self-help which
provides that:
The owner or lawful possessor of a thing has the right to exclude any person
from the enjoyment and disposal thereof. For this purpose, he may use such
force as may be reasonably necessary to repel or prevent an actual or
threatened unlawful physical invasion or usurpation of his property.
(Article 429, Civil Code)
ISSUE: Whether or not the doctrine of self-help is applicable in this case.

German Management should have filed either accion publiciana or accion


reivindicatoria to lawfully eject the farmers.
But the farmers are not the real owners and in fact, the spouses Jose have a
lawful title over the land?
Regardless of the actual condition of the title to the property, the party in
peaceable quiet possession shall not be turned out by a strong hand,
violence or terror. Further, there is now a presumption of ownership in favor
of the farmers since they are the ones occupying the said property. They can
only be ejected either by accion publicianaor accion reivindicatoria through
which the spouses Joses better right may be proven

People vs. Narvaez, 121 SCRA 389 (1983)


FACTS: Mamerto Narvaez has been convicted of murder
(qualified by treachery) of David Fleischer and Flaviano
Rubia. On August 22, 1968, Narvaez shot Fleischer and Rubia
during
the time the two were constructing a fence that would
prevent Narvaez from getting into his house and rice mill.
The defendant was taking a nap when he heard sounds of
construction and

found fence being made. He addressed the group and asked


them to stop destroying his house and asking if they could
talk things over. Fleischer responded with "No, gadamit,
proceed, go
ahead." Defendant lost his "equilibrium," and shot Fleisher
with his shotgun. He also shot Rubia who was running
towards the jeep where the deceased's gun was placed. Prior
to the
shooting, Fleischer and Co. (the company of Fleischer's
family) was involved in a legal battle with the defendant and
other land settlers of Cotabato over certain pieces of
property. At the time
of the shooting, the civil case was still pending for annulment
(settlers wanted granting of property to Fleisher and Co. to
be annulled). At time of the shooting, defendant had leased
his
property from Fleisher (though case pending and ownership
uncertain) to avoid trouble. On June 25, defendant received
letter terminating contract because he allegedly didn't pay
rent.
He was given 6 months to remove his house from the land.
Shooting was barely 2 months after letter. Defendant claims
he killed in defense of his person and property. CFI ruled that
Narvaez was guilty. Aggravating circumstances of evident
premeditation offset by the mitigating circumstance of
voluntary surrender. For both murders, CFI sentenced him to
reclusion perpetua, to indemnify the heirs, and to pay for
moral damages.

ISSUES:

1. Whether or not CFI erred in convicting defendant-appellant


despite the fact that he acted in defense of his person.
No. The courts concurred that the fencing and chiselling of
the walls of the house of the defendant was indeed a form of
aggression on the part of the victim. However, this
aggression was not done on the person of the victim but
rather on his rights to property. On the first issue, the courts
did not err. However, in consideration of the violation of
property rights, the courts referred to Art. 30 of the civil code
recognizing the right of owners to close and fence their land.
Although is not in dispute, the victim was not in the position
to subscribe to the article because his ownership of the land
being awarded by the government was still pending,
therefore putting ownership into question. It is accepted that
the victim was the original aggressor.

2. WON the court erred in convicting defendant-appellant


although he acted in defence of his rights.
Yes. However, the argument of the justifying circumstance of
self-defense is applicable only if the 3 requirements are
fulfilled. Art. 11(1) RPC enumerates these requisites:
Unlawful aggression. In the case at bar, there was unlawful
aggression towards appellant's property rights. Fleisher had
given Narvaez 6 months and he should have left him in
peace before time was up, instead of chiseling Narvaez's
house and putting up fence. Art. 536 of the Civil Code also
provides that possession may not be acquired through force
or intimidation; while Art. 539 provides that every possessor
has the right to be respected in his possession
Reasonable necessity of means employed to prevent or repel
attack. In the case, killing was disproportionate to the attack.

Lack of sufficient provocation on part of person defending


himself. Here, there was no provocation at all since he was
asleep

each group of heirs 4,000 w/o subsidiary imprisonment and


w/o award for moral damages. Appellant has already been
detained 14 years so his immediate release is ordered.

Since not all requisites present, defendant is credited with


the special mitigating circumstance of incomplete defense,
pursuant to Art. 13(6) RPC. These mitigating circumstances
are: voluntary surrender and passion and obfuscation (read
p. 405 explanation) Crime is homicide (2 counts) not murder
because treachery is not applicable on account of
provocation by the deceased. Also, assault was not
deliberately chosen with view to kill since slayer acted
instantaneously. There was also no direct evidence of
planning or preparation to kill. Art. 249 RPC: Penalty for
homicide is reclusion temporal. However, due to mitigating
circumstances and incomplete defense, it can be lowered
three degrees (Art. 64) to arrestomayor.

Gutierrez, dissenting. Defense of property can only be


invoked when coupled with form of attack on person
defending property. In the case at bar, this was not so.
Appellant should then be sentenced to prision mayor.
However, since he has served more than that, he should be
released.

3. WON he should be liable for subsidiary imprisonment since


he is unable to pay the civil indemnity due to the offended
party.
No. He is not liable to be subsidiarily imprisoned for
nonpayment of civil indemnity. RA 5465 made the provisions
of Art. 39 applicable to fines only and not to reparation of
damage caused, indemnification of consequential damages
and costs of proceedings. Although it was enacted only after
its conviction, considering that RA 5465 is favorable to the
accused who is not a habitual delinquent, it may be given
retroactive effect pursuant to Art. 22 of the RPC.
Judgment: Defendant guilty of homicide but w/ mitigating
circumstances and extenuating circumstance of incomplete
self defense. Penalty is 4 months arresto mayor and to
indemnify

Villafuerte vs CA

Issue:
Whether or not the ordinance made by Quezon City is a valid
taking of private property
Ruling:

City Government of Quezon vs. Judge Ericta G


R No. L-34915 June 24, 1983

Facts:
An ordinance was promulgated in Quezon city which
approved the the regulation ofestablishment of private
cemeteries in the said city. According to the ordinance, 6% of
the total area of the private memorial park shall be set aside
for charity burial of deceased persons who are paupers and
have been residents of QC. Himlayang Pilipino, a private
memorial park, contends that the taking or confiscation of
property restricts the use of property such that it cannot be
used for any reasonable purpose and deprives the owner of
all beneficial use of his property. It also contends that the
taking is not a valid exercise of police power, since the
properties taken in the exercise of police power are
destroyed and not for the benefit of the public.

No, the ordinance made by Quezon City is not a valid way of


taking private property. The ordinace is actually a taking
without compensation of a certain area from a private
cemetery to benefit paupers who are charges of the
municipal corporation. Instead of building or maintaing a
public cemeteries. State's exercise of the power of
expropriation requires payment of just compensation. Passing
the ordinance without benefiting the owner of the property
with just compensation or due process, would amount to
unjust taking of a real property. Since the property that is
needed to be taken will be used for the public's benefit, then
the power of the state to expropriate will come forward and
not the police power of the state.

Spouses Custodio vs. CA


Spouses Cristino and Brigida Custodio and Spouses Lito and
Maria Cristina Santos vs. Court of Appeals, Heirs of Pacifico C.
Mabasa
G.R. No. 116100, February 9, 1996
Regalado, J.:
Doctrine: Every owner has an absolute right over his property and his
act of fencing and enclosing the same was an act which he may
lawfully perform in the employment and exercise of said right.
Whatever injury or damage that may have been sustained by others by
reason of the rightful use of the said land by the owner is damnum
absque injuria.

Facts: The respondent (Pacifico Mabasa) owns a parcel of land with a

Trial court rendered a decision ordering the Petitioners Custodios and

two-door apartment erected thereon situated at Interior P. Burgos St.,

Santoses to give Respondent Mabasa permanent access ingress and

Palingon, Tipas, Tagig, Metro Manila. Said property may be described to

egress, to the public street and Mabasa to pay the Custodios and

be surrounded by other immovables pertaining to respondents herein.

Santoses the sum of Eight Thousand Pesos (P8,000) as indemnity for

As an access to P. Burgos Street from respondents property, there are

the permanent use of the passageway.

two possible passageways. The first passageway is approximately one


meter wide and is about 20 meters distan(t) from Mabasas residence

Respondent Mabasa went to the CA raising the sole issue of whether or

to P. Burgos Street. Such path is passing in between the previously

not the lower court erred in not awarding damages in their favor. The

mentioned row of houses of the petitioners The second passageway is

CA rendered its decision affirming the judgment of the trial court with

about 3 meters in width and length from Mabasas residence to P.

modification only insofar as the. grant of damages to Mabasa The

Burgos Street; it is about 26 meters. In passing thru said passageway,

motion for reconsideration filed by the petitioners was denied.

a less than a meter wide path through the septic tank and with 5-6
meters in length, has to be traversed.

Issues:
Whether the grant of right of way to herein private respondent Mabasa

When said property was purchased by Mabasa, there were tenants

is proper.

occupying the remises and who were acknowledged by Mabasa as

Whether the award of damages is in order.

tenants. However, sometime in February, 1982, one of said tenants

Held:

vacated the apartment and when Mabasa went to see the premises, he

No. Herein petitioners are already barred from raising the same.

saw that there had been built an adobe fence in the first passageway

Petitioners did not appeal from the decision of the court a quo granting

making it narrower in width. Said adobe fence was first constructed by

private respondents the right of way, hence they are presumed to be

Petitioners Santoses along their property which is also along the first

satisfied with the adjudication therein. With the finality of the judgment

passageway. Petitioner Morato constructed her adobe fence and even

of the trial court as to petitioners, the issue of propriety of the grant of

extended said fence in such a way that the entire passageway was

right of way has already been laid to rest.

enclosed. And it was then that the remaining tenants of said apartment

No. A reading of the decision of the CA will show that the award of

vacated the area.

damages was based solely on the fact that the original plaintiff,
Pacifico Mabasa, incurred losses in the form of unrealized rentals when

Petitioner Ma. Cristina Santos testified that she constructed said fence

the tenants vacated the leased premises by reason of the closure of

because of some other inconveniences of having (at) the front of her

the passageway.However, the mere fact that the plaintiff suffered

house a pathway such as when some of the tenants were drunk and

losses does not give rise to a right to recover damages.

would bang their doors and windows.


There is a material distinction between damages and injury. Injury is
the illegal invasion of a legal right; damage is the loss, hurt, or harm

which results from the injury; and damages are the recompense or

sustained by private respondents by reason of the rightful use of the

compensation awarded for the damage suffered. Thus, there can be

said land by petitioners is damnum absque injuria.

damage without injury in those instances in which the loss or harm was
not the result of a violation of a legal duty. (damnum absque injuria). In

Caveat: Anyone who claims this digest as his own without

order that a plaintiff may maintain an action for the injuries of which he

proper authority shall be held liable under the law of Karma.

complains, he must establish that such injuries resulted from a breach


of duty which the defendant owed to the plaintiff a concurrence of
injury to the plaintiff and legal responsibility by the person causing it
(damnum et injuria.)

PROPERTY DIGEST PART 4


Accession Cases

In the case at bar, although there was damage, there was no legal
injury. The act of petitioners in constructing a fence within their lot is a
valid exercise of their right as owners, hence not contrary to morals,
good customs or public policy. The law recognizes in the owner the
right to enjoy and dispose of a thing, without other limitations than
those established by law. It is within the right of petitioners, as owners,
to enclose and fence their property. Article 430 of the Civil Code
provides that (e)very owner may enclose or fence his land or
tenements by means of walls, ditches, live or dead hedges, or by any
other means without detriment to servitudes constituted thereon.

At the time of the construction of the fence, the lot was not subject to
any servitudes. There was no easement of way existing in favor of
private respondents, either by law or by contract. The fact that private
respondents had no existing right over the said passageway is
confirmed by the very decision of the trial court granting a compulsory
right of way in their favor after payment of just compensation.

Hence, prior to said decision, petitioners had an absolute right over


their property and their act of fencing and enclosing the same was an
act which they may lawfully perform in the employment and exercise
of said right. To repeat, whatever injury or damage may have been

Pacific farms Inc vs Esguerra 30 scra 684


(1969)
Facts:
Carried Lumber Company sold and delivered lumber and
construction materials to the Insular Farms, Inc. which the
latter used in the construction of the aforementioned six
buildings at its compound. However, the value of the
materials was not paid by Insular Farms, Inc. Company
instituted civil case with the Court of First Instance of
Pangasinan to recover the said unpaid balance from the
Insular Farms, Inc. Trial court rendered judgment sustaining
the Company's claim. The judgment debtor did not appeal; so
the corresponding writ of execution was issued. Pacific Farms,
Inc. filed a third party claim, asserting ownership over levied
buildings which it acquired from Insular Farms. Thereafter,
sheriff proceeded with public auction. Meanwhile, Pacific
Farms filed a complaint against the Company and the sheriff
with the court a quo, praying that judgment be rendered, (a)
declaring null and void the levy and judicial sale of the six
buildings, and (b) adjudging the defendants jointly and
severally liable to the plaintiff in the sum of P2,000 by way of

actual damages and for such amount as the court may deem
proper and just to impose by way of exemplary damages and
for costs of the suit. Trial Court annulled the levy but denied
claim for actual and exemplary damages.
Issue: Whether or not the appellant acted correctly in
bringing an action against the Insular Farms, Inc. and
enforcing its right of reimbursement.
Held:
Yes, the appellant acted correctly in bringing an action (D775) against the Insular Farms, Inc. and enforcing its right of
reimbursement through the execution of the final judgment it
obtained in the said case against the six buildings in the
possession of the appellee who now stands to benefit
therefrom. It follows, as a necessary corollary, that the sale
at public auction conducted by the defendant sheriff of the
six buildings described in the certificate of sale dated
February12, 1962, exhibit 7, was valid and effective. Also, the
application by analogy of the rules of accession would suffice
for a just adjudication. Article 447 of the Civil Code provides:
The owner of the land who makes thereon personally or
through another, plantings, constructions or works with the
materials of another, shall pay their value; and, if he acted in
bad faith, he shall also be obliged to the reparation of
damages. The owner of the materials shall have the right to
remove them only in case he can do so without injury to the
work constructed, or without the plantings, constructions or
works being destroyed. However, if the landowner acted in
bad faith, the owner of the materials may remove them in
any event with a right to be indemnified for damages.
The above quoted legal provision contemplated a principal
and an accessory, the land being considered the principal,
and the plantings, costructions or works, the accessory. The
owner of the land who in good faith whether personally or
through another- makes constructions or works thereon,

using materials belonging to somebody else, becomes the


owner of the said materials with the obligation however of
praying for their value. The owner of the materials, on the
other hand, is entitled to recover them, provided no
substantial injury is caused to the landowner. Otherwise, he
has the right to reimbursement for the value of his materials.

Tan Queto v. CA [G.R. No. L-35648. February 27,


1987.];
Facts:
Restituta Tagalinar Guangco de Pombuena received the
questioned lot (Lot 304-B of the Cadastre Survey of the
Municipality of Centro, Misamis Occidental) either as a
purported donation or by way of purchase on 11 February
1927 for P50.00 as the alleged consideration thereof. The
transaction took place during her mothers lifetime (her
father having predeceased the mother) and consummated
while Restituta was already married to her husband Juan
Pombuena. On 22 January 1935, Juan filed an application of
Torrens title over the land for himself and his supposed coowner Restituta. On 22 November 1938, a decision was
promulgated (GLRC 1638, Cadastral Case 12) pronouncing
Juan (married to Restituto) as the owner of the land. On 22
September 1949 a contract of lease over the lot was entered
into between Pershing Tan Queto and Restituta (with the
consent of her husband) for a period of 10 years.
Meanwhile, On 27 December 1960 Restituta sued Tan Queto
for unlawful detainer (the lease contract having expired)
before the Municipal Court of Ozamis City.
On 22 April 1962, as a consequence of the cadastral case, an
OCT was issued in Juans name. On 10 October 1962, Tan
Queto and Juan entered into a barter agreement whereby Tan

Queto became the owner of the disputed lot, and the


spouses in turn became the owners of a parcel of land with
the house constructed thereon previously owned (that is,
before the barter) by Tan Queto. Thereafter, Tan Queto
constructed on the disputed land a concrete building, without
any objection on the part of Restituta.
The Municipal court ruled in favor of the spouses in the
unlawful detainer case; but on appeal in the CFI, the entire
case was dismissed because of an understanding (barter)
entered into by Juan and Tan Queto.
Restituta sued both Juan and Tan Queto for reconveyance of
the title over the registered but disputed lot, for annulment
of the barter, and for recovery of the land with damages. The
CFI and the Court of Appeals found the disputed lot as
paraphernal and that Tan Queto was a builder in bad faith.
These findings were regarded by the Supreme Court as
findings of facts and thus ordinarily conclusive upon the
Court. Tan Queto filed for a motion for reconsideration of the
Supreme Court decision dated 16 May 1983.

The Supreme Court set aside its decision promulgated on 16


May 1983, and rendered a new one declaring the questioned
lot together with the building thereon, as Tan Quetos
exclusive property; without costs.
1. Findings of the lower courts ordinary conclusive upon the
Court; exception, if erroneous
The findings of the Court of First Instance and the Court of
Appeals were regarded by the Supreme Court as findings of
facts and thus ordinarily conclusive upon the Court.
Assuming they are factual findings, still if they are erroneous
inferences from certain facts, they cannot bind the Court.

2. Land not transferred to Restituta by donation, for it to be


paraphernal
The oral donation of the lot cannot be a valid donation intervivos because it was not executed in a public instrument (Art.
749, Civil Code), nor as a valid donation mortis causa for the
formalities of a will were not complied with. The allegation
that the transfer was a conveyance to Restituta of her
hereditary share in the estate of her mother (or parents)
cannot be sustained for the contractual transmission of
future inheritance is generally prohibited.
3. Land is conjugal, not paraphernal; Ownership by tradition
The land is conjugal, not paraphernal. Ownership was
acquired by the spouses by tradition (delivery) as a
consequence of the contract of sale (See Art. 712, Civil Code)
with P50.00 (then a considerable amount) as the cause or
consideration of the transaction. The lot is therefore conjugal,
having been acquired by the spouses thru onerous title (the
money used being presumably conjugal, there being no proof
that Restituta had paraphernal funds of her own).
4. Sale not fictitious nor simulated; Allegation of simulation
cannot prejudice a stranger

The sale cannot be said to be fictitious or simulated (and


therefore void) as there was a valid consideration therefor.
Assuming that there had indeed been a simulation, the
parties thereto cannot use said simulation to prejudice a
stranger to said strategem (like petitioner herein).
5. Tan Queto recognized Restituta as an owner, not the owner
Tan Queto admitted Restituta was an owner (not the
owner) of the lot in his Answer, and this is true, for she was a

co-owner (with Juan, and therefore an owner.) There is no


admission of Restitutas exclusive ownership.
6. Assuming Tan Queto recognized Restituta as the owner;
bad faith of one neutralizes the bad faith of the other
Even assuming that despite registration of the lot as
conjugal, Tan Queto nursed the belief that the lot was
actually Restitutas (making him in bad faith), still Restitutas
failure to prohibit him from building despite her knowledge
that construction was actually being done, makes her also in
bad faith. The net resultant of mutual bad faith would entitle
Tan Qyeto to the rights of a builder in good faith (Art. 448,
Civil Code), ergo, reimbursement should be given him if
Restituta decides to appropriate the building for herself (Art.
448, Civil Code).
7. Tan Queto an owner-possessor
Tan Queto having bartered his own lot and small house with
the questioned lot with Juan (who has been adverted to by a
court decision and by the OCT a conjugal owner) may be said
to be the owner-possessor of the lot. Certainly he is not
merely a possessor or builder in good faith (this phrase
presupposes ownership in another); much less is he a builder
in bad faith. He is a builder-possessor (jus possidendi)
because he is the owner himself.
8. Jus possessionis, jus possidendi; good faith and bad faith
The Chapter on Possession (jus possessionis, not jus
possidendi) in the Civil Code refers to a possessor other than
the owner. The difference between a builder (or possessor) in
good faith and one in bad faith is that the former is not aware
of the defect or flaw in his title or mode of acquisition while
the latter is aware of such defect or flaw (Art. 526, Civil
Code). But in either case there is a flaw or defect. In the
present case, there is no such flaw or defect because it is Tan

Queto himself (not somebody else) who is the owner of the


property.

Pleasantville Development v. CA
[G.R. No. 79688. February 1, 1996.]
Facts:
Edith Robillo purchased from Pleasantville Development
Corporation a parcel of land designated as Lot 9, Phase II and
located at Taculing Road, Pleasantville Subdivision, Bacolod
City. Eldred Jardinico bought the rights to the lot from Robillo.
At that time, Lot 9 was vacant. On the other hand, on 26
March 1974, Kee bought on installment Lot 8 of the same
subdivision from C.T. Torres Enterprises, Inc. (CTTEI), the
exclusive real estate agent of Pleasantville Development.
Under the Contract to Sell on Installment, Kee could possess
the lot even before the completion of all installment
payments.
On 20 January 1975, Kee paid CTTEI the relocation fee of
P50.00 and another P50.00 on 27 January 1975, for the
preparation of the lot plan. These amounts were paid prior to
Kees taking actual possession of Lot 8. After the preparation
of the lot plan and a copy thereof to Kee, CTTEI through its
employee, Zenaida Octaviano, accompanied Kees wife,
Donabelle Kee, to inspect Lot 8. Unfortunately, the parcel of
land pointed by Octaviano was Lot 9. Thereafter, Kee
proceeded to construct his residence, a store, an auto repair
shop and other improvements on the lot. Upon completing all
payments, Jardinico secured on 19 December 1978 from the
Register of Deeds of Bacolod City TCT 106367 in his name. It
was then that he discovered that improvements had been
introduced on Lot 9 by Wilson Kee, who had taken possession
thereof. After the discovery, Jardinico confronted Kee. The

parties tried to reach an amicable settlement, but failed. On


30 January 1981, Jardinicos lawyer wrote Kee, demanding
that the latter remove all improvements and vacate Lot 9.
When Kee refused to vacate, Jardinico filed with the MTCC a
complaint for ejectment with damages against Kee. Kee, in
turn, filed a third-party complaint against Pleasantville and
CTTEI. The MTCC held that the erroneous delivery of Lot 9 to
Kee was attributable to CTTEI. The MTCC also found that
Pleasantville had already rescinded its contract with Kee over
Lot 8 for the latters failure to pay the installments due, and
that Kee had not contested the rescission. The rescission was
effected in 1979, before the complaint was instituted.
The MTCC concluded that Kee no longer had any right over
the lot subject of the contract between him and Pleasantville.
Consequently, Kee must pay reasonable rentals for the use of
Lot 9, and, furthermore, he cannot claim reimbursement for
the improvements he introduced on said lot. The MTCC thus
ordered Kee to vacate Lot 9, to remove all structures and
improvements he introduced thereon, and to pay the
Jardinico rentals of P15.00 a day computed from the time the
suit was filed on 12 March 1981 Property, 2003 ( 156 )
Haystacks (Berne Guerrero) until he vacateds the premises;
such amount bearing an interest of 12% per annum. The
MTCC also ordered CTTI and Pleasantville to pay Jardinico in
solidum for the amount of P3,000 as attorneys fees and
P700 as cost and litigation expenses. On appeal, the RTC
Bacolod City (Branch 48) ruled that Pleasantville and CTTEI
were not at fault or were not negligent and found Kee a
builder in bad faith. Thus, the appellate court affirmed the
decision with respect to the order to vacate the premises of
Lot 9, the removal of the structure and improvements
introduced thereon at Kees expense, and to pay a rental of
P15.00 a day until he vacates the premises, with an interest
of 12% per annum. The Court further rendered judgment
against Kee to pay Jardinico the sum of P3,000.00 as
attorneys fees, plus costs of litigation; dismissed the third-

party complaint against Pleasantville CTTEI, and reversed the


order Pleasantville and CTTEI to pay Jardinico attorneys fees
and costs of litigation. Following the denial of his motion for
reconsideration on 20 October 1986, Kee appealed directly to
the Supreme Court, which referred the matter to the Court of
Appeals. Pending resolution of the case before the Court of
Appeals, Jardinico and Kee on 24 July 1987 entered into a
deed of sale, wherein the former sold Lot 9 to Kee. Jardinico
and Kee did not inform the Court of Appeals of such deal. The
appellate court ruled that Kee was a builder in good faith
(entitled to rights under Articles 448, 546 and 548 of the Civil
Code), as he was unaware of the mix-up when he began
construction of the improvements on Lot 8. It further ruled
that the erroneous delivery was due to the negligence of
CTTEI, and that such wrong delivery was likewise imputable
to its principal, Pleasantville; and thus ordered the CTTEI and
Pleasantville to be solidarily liable for the demolition
expenses and value of improvements destroyed or rendered
useless in case Jardinico decides to appropriate the
improvements and thereafter remove the structures; or for
the amount representing the value of Lot 9 that Kee should
pay to Jardinico if Jardinico chose to sell the land to Kee. The
appellate court ordered CTTEI and Pleasantville to pay in
solidum the amount of P3,000.00 to Jardinico as attorneys
fees, as well as litigation expenses; and ruled that the award
of rentals was without basis. Further, the appellate court
remanded the case to the court of origin for the
determination of the actual value of the improvements and
the property (Lot 9). Pleasantville filed the petition for review
on certiorari. The Supreme Court partially granted the
petition, and modified the decision of the Court of Appeals by
declaring Wilson Kee a builder in good faith; and that
Pleasantville Development and C.T. Torres Enterprises
solidarily liable for damages due to negligence (however,
since the amount and/or extent of such damages was proven
during the trial, the same cannot now be quantified and

awarded). The Court also ordered Pleasantville Development


and C.T. Torres Enterprises to pay in solidum the amount of
P3,000.00 to Jardinico as attorneys fees, as well as litigation
expenses. The Court dispensed with the award of rentals to
Jardinico.
1. Kee a builder in good faith; Prudent acts to ascertain land
to build on Under the Torrens system of land registration, Kee
is presumed to have knowledge of the metes and bounds of
the property with which he is dealing. But as Kee is a layman
not versed in the technical description of his property, he had
to find a way to ascertain that what was described in TCT
69561 matched Lot 8. Thus, he went to the subdivision
developers agent and applied and paid for the relocation of
the lot, as well as for the production of a lot plan by CTTEIs
geodetic engineer. Upon Kees receipt of the map, his wife
went to the subdivision site accompanied by CTTEIs
employee, Octaviano, who authoritatively declared that the
land she was pointing to was indeed Lot 8. Having full faith
and confidence in the reputation of CTTEI, and because of the
companys positive identification of the property, Kee saw no
reason to suspect that there had been a misdelivery. The
steps Kee had taken to protect his interests were reasonable.
There was no need for him to have acted ex-abundantia
cautela. such as being present during the geodetic
engineers relocation survey or hiring an independent
geodetic engineer to countercheck for errors, for the final
delivery of subdivision lots to their owners is part of the
regular course of everyday business of CTTEI. Because of
Property, 2003 ( 157 ) Haystacks (Berne Guerrero) CTTEIs
blunder, what Kee had hoped to forestall did in fact transpire.
Kee had acted in the manner of a prudent man in
ascertaining the identity of his property.
2. Scenario of bad faith improbable; Good faith presumed It is
thus highly improbable that a purchaser of a lot would
knowingly and willingly build his residence on a lot owned by

another, deliberately exposing himself and his family to the


risk of being ejected from the land and losing all
improvements thereon, not to mention the social humiliation
that would follow. Good faith consists in the belief of the
builder that the land he is building on is his and his ignorance
of any defect or flaw in his title. And as good faith is
presumed, the one alleging bad faith has the burden of
proving bad faith.
3. Contractual breach cannot be the basis to negate the
presumption of builder in good faith Violations of paragraphs
22 and 26 of the Contract of Sale on Installment have no
bearing whatsoever on whether Kee was a builder in good
faith, i.e. on his state of mind at the time he built the
improvements on Lot 9. These alleged violations may give
rise to Pleasantvilles cause of action against Kee under the
said contract (contractual breach), but may not be bases to
negate the presumption that Kee was a builder in good faith.
4. Rescission does not negate the negligence of CTTEI The
Contract of Sale on Installment covering Lot 8 between
Pleasantville and Kee, which was rescinded long before the
present action was instituted, has no relevance on the
liability of Pleasantville, as such fact does not negate the
negligence of its agent in pointing out the wrong lot to Kee.
Such circumstance is relevant only as it gives Jardinico a
cause of action for unlawful detainer against Kee.
5. Recovery of damages not waived Kee did not contracted
away his right to recover damages resulting from
Pleasantvilles negligence. Such waiver would be contrary to
public policy and cannot be allowed. Rights may be waived,
unless the waiver is contrary to law, public order, public
policy, morals, or good customs, or prejudicial to a third
person with a right recognized by law.
6. Principal responsible for acts of agent if damaged caused
to third persons; Agent is personally liable for damages if he

exceeds his authority The principal is responsible for the acts


of the agent, done within the scope of his authority, and
should bear the damage caused to third persons. On the
other hand, the agent who exceeds his authority is personally
liable for the damages. In the present case, CTTEI was acting
within its authority as the sole real estate representative of
Pleasantville when it made the delivery to Kee. In acting
within its scope of authority, it was, however, negligent. It is
this negligence that is the basis of Pleasantvilles liability, as
principal of CTTEI, per Articles 1909 and 1910 of the Civil
Code.
7. Deed of Sale between Kee and Jardinico merely regulates
the reciprocal rights of the parties and has no effect on the
liability of Pleasantville The deed of sale regulates the
reciprocal rights of Kee and Jardinico; it stressed that they
had reached an agreement independent of the outcome of
the case. The terms and conditions in the said deed of sale
are strictly for the parties thereto and that there is no
waiver made by either of the parties in said deed of whatever
favorable judgment or award the Court of Appeals may make
in Kees and Jardinicos favor against Pleasantville and
CTTEI. The deed of sale can have no effect on the liability of
Pleasantville. Pleasantvilles liability is grounded on the
negligence of its agent.
8. Pleasantville supposedly liable for damages due to agents
negligence; Due to lack of evidence, no damages due
Pleasantvilles liability lies in the negligence of its agent
CTTEI. For such negligence, Pleasantvilles should be held
liable for damages. The extent and/or amount of damages to
be awarded is a factual issue which should be determined
after evidence is adduced. However, there is no showing that
such evidence was Property, 2003 ( 158 ) Haystacks (Berne
Guerrero) actually presented in the trial court; hence no
damages could be awarded.

9. Appellate court erred in modification of the application of


the law on ground of equity The rights of Kee and Jardinico
vis-a-vis each other, as builder in good faith and owner in
good faith, respectively, are regulated by law (i.e., Arts. 448,
546 and 548 of the Civil Code). It was error for the Court of
Appeals to make a slight modification in the application of
such law, on the ground of equity. At any rate, Kee and
Jardinico have amicably settled through their deed of sale
their rights and obligations with regards to Lot 9. Thus, the
Court deleted the dispositive portion of the Court of Appeals
Decision holding Pleasantville and CTTEI solidarily liable for
demolition expenses or the amount pertaining to the value of
the lot, whichever is applicable in the exercise of the
landowners options.
10. Award of attorneys fees lies with the discretion of the
court depending on the cases circumstances The award of
attorneys fees lies within the discretion of the court and
depends upon the circumstances of each case. The Supreme
Court shall not interfere with the discretion of the Court of
Appeals. Jardinico was compelled to litigate for the protection
of his interests and for the recovery of damages sustained as
a result of the negligence of Pleasantviles agent.
11. No need to remand the case for the determination of the
value and the land In view of the deed of sale entered into by
Kee and Jardinico, which deed governs the rights of Jardinico
and Kee as to each other, there is also no further need to
remand the case to the court of origin for determination of
the actual value of the improvements and the property (Lot
9), as well as for further proceedings in conformity with
Article 448 of the New Civil Code.

Baltazar v. Caridad
[G.R. No. L-23509. June 23, 1966.]

Facts: In the cadastral proceeding (Cadastral Case 54, GLRO


Cadastral Case 1222), the trial court rendered decision,
dated 23 January 1941, awarding Lot 8864 of the Laoag
(Ilocos Norte) cadastre to the spouses Julio Baltazar and
Constancia Valencia as their conjugal partnership property.
Said decision having become final, the corresponding decree
was issued on 12 July 1941, and pursuant thereto, said lot
was registered in the names of applicant spouses under OCT
O-1445, which was later transcribed, on 5 November 1959, in
the office of the Register of Deeds of Ilocos Norte. In the
meanwhile, Julio Baltazar, the registered owner of Lot 8864,
died. On 6 December 1961, his surviving Property, 2003
( 19 ) Haystacks (Berne Guerrero) wife and children filed a
motion, in the cadastral case praying for writ of possession
against Silvina Caridad and her daughter, Eduarda Caridad,
who had been in possession of the southern portion of said
Lot 8864 since 1939, while the cadastral case involving said
lot was pending before the trial court, and before the
decision was rendered and the corresponding decree issued
in 1941. No writ having theretofore been issued in Baltazars
favor, the trial court issued an order, on 11 December 1961,
granting Baltazars motion, and overruled Caridads
opposition but directed the sheriff not to remove or destroy
the permanent improvements on the lot without an express
command. On 2 January 1962, the order having become final,
the sheriff enforced the writ and placed Baltazar in
possession of the southern portion of the lot. On 23 January
1962, Baltazar presented a motion to compel Eduarda and
Silvina Caridad to remove their respective houses which they
built in 1958 and 1959, respectively, in the southern portion
of the disputed lot, and, in the event of their failure to do so,
to order the sheriff to demolish the same. On 20 March 1962,
the trial court, after due hearing, granted Baltazars motion,
ordering the Caridads to remove their respective houses from
the southern portion of said lot 8864 within 30 days from
receipt of said order. Not satisfied, the Caridads appealed

(CA-GR 31289-R). The appellate court, however, certified the


appeal to the Supreme Court for raising only questions of law.
The Supreme Court affirmed the appealed order; with costs
against the Caridads. 1. Order compelling Caridads to remove
their respective houses from the disputed lot; Lack of
opposition in previous proceedings The order, dated 20
March 1962, of the cadastral court, granting Baltazars
motion to compel the Caridads to remove their respective
houses from the disputed lot, is valid and enforceable against
the latter. This may be concluded based on the
circumstances that (1) that the Caridads do not dispute that
during the pendency of the cadastral proceeding (to which
judgment was rendered awarding said lot 8864, and
consequent issuance of the final decree of registration of the
same, in favor of Julio Baltazar), the late Andres Caridad, his
surviving spouse Silvina Caridad, and their children, one of
whom is Eduarda Caridad, were in possession of the southern
portion of the disputed lot ; (2) that Eduarda Caridad claims
right and title thereto as a mere heir and successor-ininterest of said Andres Caridad; and (3) that the Caridads do
not dispute the propriety and validity of the order of the
cadastral court, granting the writ of possession in favor of
Baltazar as well as its enforcement. 2. Jurisdiction of the CFI,
sitting as a land registration court, to issue writ of possession
and order demolition of improvements In Marcelo vs.
Mencias, etc., et al. (L-15609, 29 April 1960, 58 OG 3349),
the Court had already upheld the jurisdiction or authority of
the CFI, sitting as a land registration court, to order, as a
consequence of the writ of possession issued by it, the
demolition of improvements introduced by the successor-ininterest of a defeated oppositor in the land registration case.
3. Section 13, Rule 39 of the Rules of Court Section 13, Rule
39 of the Rules of Court (How execution for the delivery or
restitution of property enforced), provides that the officer
must enforce an execution for the delivery or restitution of
property by placing the plaintiff in possession of such

property, and by levying as hereinafter provided upon so


much of the property of the judgment debtor as will satisfy
the amount of costs, damages, rents, and profits included in
the execution. However, the officer shall not destroy,
demolish or remove the improvements made by the
defendant or his agent on the property, except by special
order of the court, which order may only issue upon petition
of the plaintiff after due hearing and upon the defendants
failure to remove the improvements within a reasonable time
to be fixed by the court. 4. Rule applies to land registration
cases in a suppletory character and not only to ordinary
actions involving the delivery or restitution of property
Property, 2003 ( 20 ) Haystacks (Berne Guerrero) The
provision of the Rules of Court applies not only to ordinary
actions involving the delivery or restitution of property, but
also to proceedings under the land registration law. The
provisions of the Rules of Court are applicable to land
registration cases in a suppletory character (Rule 132). Thus,
if the writ of possession issued in a land registration
proceeding implies the delivery of possession of the land to
the successful litigant therein (Demorar vs. Ibaez, 51 OG
2872, Pasay Estate Company vs. Del Rosario, et al., 11 Phil.
391; Manlapas vs. Llorente, 48 Phil. 298), a writ of demolition
must, likewise, issue, especially considering that the latter
writ is but a complement of the former which without said
writ of demolition would be ineffective. To require a
successful litigant in a land registration case to institute
another action for the purpose of obtaining possession of the
land adjudged to him, or for the purpose of securing fruits of
his victory, would be a cumbersome process. It would foster
unnecessary and expensive litigations and result in
multiplicity of suits, which the judicial system abhors. 5.
Power of a judge to issue all auxiliary writs to carry into effect
the jurisdiction conferred upon the court by law Section 6,
Rule 124, of the Rules of Court provides that when by law
jurisdiction is conferred on a court or judicial officer, all

auxiliary writs, processes and other means necessary to


carry it into effect may be employed by such court or officer;
and if the procedure to be followed in the exercise of such
jurisdiction is not specifically pointed out by these rules, any
suitable process or mode of proceeding may be adopted
which appears most conformable to the spirit of said rules.
In the present case, the Judge has the power to issue all
auxiliary writs, including the writ of demolition, processes
and other means necessary to carry into effect the
jurisdiction conferred upon it by law in land registration cases
to issue a writ of possession to the successful litigant. 6.
Every court has inherent power to do necessary thing s for
the administration of justice In Shioji vs. Harvey, 43 Phil 333,
it was pointed out that independent of any statutory
provision, every court has inherent power to do all things
reasonably necessary for the administration of justice within
the scope of its jurisdiction. In line with this doctrine, the
Judge has the inherent power to issue the writ of demolition.
Its issuance is reasonably necessary to do justice to
petitioner who is being deprived of the possession of the lots
in question, by reason of the continued refusal of the
respondent to remove his house thereon and restore
possession of the premises to petitioner. 7. Builders in bad
faith; Rights of registered owner cannot be defeated by an
unsuccessful opponent through the subterfuge of replacing
his old house with a new one from time to time The Caridads
cannot be regarded as builders in good faith because they
are bound by the 1941 decree of registration that obligated
their parents and predecessors-in-interest. Good faith must
rest on a colorable right in the builder, beyond a mere
stubborn belief in ones title despite judicial adjudication. The
fact that in 1959 the Caridads demolished and replaced their
old house with new and bigger ones cannot enervate the
rights of the registered owners. Otherwise, the rights of the
latter to enjoy full possession of their registered property
could be indefinitely defeated by an unsuccessful opponent

through the simple subterfuge of replacing his old house with


a new one from time to time

Geminiano v. CA
[G.R. No. 120303. July 24, 1996.]
Facts: Lot 3765-B-1 (314 sq. m.) was originally owned by
Paulina Amado vda. de Geminiano, the mother of Federico,
Maria, Ernesto, Asuncion, Larry and Marlyn Geminiano. On a
12-sq. m. portion of that lot stood the Geminianos unfinished
bungalow, which the Geminianos sold in November 1978 to
Dominador and Mary Nicolas for the sum of P6,000.00, with
an alleged promise to sell to the latter that portion of the lot
occupied by the house. Subsequently, Paulina AmadoGeminiano executed a contract of lease over a 126 sq. m.
portion of the lot, including that portion on which the house
stood, in favor of the Nicolas spouse for P40 per month for a
period of 7 years commencing on 15 November 1978. The
Nicolas spouses then introduced additional improvements
and registered the house in their names. After the expiration
of the lease contract in November 1985, however, the
Paulina refused to accept the monthly rentals. It turned out
that the lot in question was the subject of a suit, which
resulted in its acquisition by one Maria Lee in 1972. In 1982,
Lee sold the lot to Lily Salcedo, who in turn sold it in 1984 to
the spouses Agustin and Ester Dionisio. On 14 February 1992,
the Dionisio spouses executed a Deed of Quitclaim over the
said property in favor of the Geminianos. As such, the lot was
registered in the latters names. On 9 February 1993, the
Geminianos sent, via registered mail, a letter addressed to
Mary Nicolas demanding that she vacate the premises and
pay the rentals in arrears within 20 days from notice. Upon
failure of the Nicolas spouses to heed the demand, the
Geminianos filed with the MTCC of Dagupan City a complaint
for unlawful detainer and damages. The trial court held that
there was no lease to speak of to be renewed as the lot was

acquired by Maria Lee in 1972, and that if indeed there is a


legal lease existing, its renewal can only be made on a
month-to-month pursuant to Article 1687 of the Civil Code;
that the lessees were not builders in good faith and the
reimbursement as such are governed by Article 1678; and
that the value of the house and improvements was P180,000
as there was controverting evidence presented. The Court
thus ordered the Nicolas spouses to vacate the premises, to
pay the Geminianos P40 a month as reasonable
compensation for their stay thereon from the filing of the
complaint on 14 April 1993 until they vacated, and to pay the
sum of P1,000 as attorneys fees, plus costs. On appeal by
the Nicolas spouses, the RTC Dagupan City reversed the trial
courts decision and rendered a Property, 2003 ( 69 )
Haystacks (Berne Guerrero) new judgment: (1) ordering the
Geminianos to reimburse the Nicolas spouses for the value of
the house and improvements in the amount of P180,000.00
and to pay the latter P10,000.00 as attorneys fees and
P2,000.00 as litigation expenses; and (2) allowing the Nicolas
spouses to remain in possession of the premises until they
were fully reimbursed for the value of the house. It ruled that
since the Nicolas spouses were assured by the Geminianos
that the lot they leased would eventually be sold to them,
they could be considered builders in good faith, and as such,
were entitled to reimbursement of the value of the house and
improvements with the right of retention until reimbursement
had been made. On appeal, this time by Geminianos, the
Court of Appeals affirmed the decision of the RTC and denied
the Geminianos motion for reconsideration. Hence, the
petition for review on certiorari. The Supreme Court granted
the petition; reversing and setting aside the decision of the
Court of Appeals of 27 January 1995 in CA-GR SP 34337; and
reinstating the decision of Branch 3 of the Municipal Trial
Court in Cities of Dagupan City in Civil Case 9214; with costs
against the Nicolas spouses. 1. Non-owner of the premises
may lease property While the right to lease property is an

incident of title and possession, a person may be a lessor and


occupy the position of a landlord to the tenant although he is
not the owner of the premises leased. After all, ownership of
the property is not being transferred, only the temporary use
and enjoyment thereof. 2. Nicolas spouses estopped;
Estoppel applies even if lessor has no title, may be asserted
not only by original lessor but also those who succeed to his
title The Nicolas spouses came into possession of a 126 sq.
m. portion of the said lot by virtue of a contract of lease
executed by the Geminianos mother in their favor. The
juridical relation between the Geminianos mother as lessor,
and the Nicolas spouses as lessees, is therefore wellestablished, and carries with it a recognition of the lessors
title. The lessees who had undisturbed possession for the
entire term under the lease, are then estopped to deny their
landlords title, or to assert a better title not only in
themselves, but also in some third person while they remain
in possession of the leased premises and until they surrender
possession to the landlord. This estoppel applies even though
the lessor had no title at the time the relation of lessor and
lessee was created, and may be asserted not only by the
original lessor, but also by those who succeed to his title. 3.
Lessees not possessors not builders in good faith Being mere
lessees, the Nicolas spouses knew that their occupation of
the premises would continue only for the life of the lease.
Plainly, they cannot be considered as possessors nor builders
in good faith. 4. Article 448 in relation to Article 546 applies
only to a possessor in good faith; does not apply to lessee
Article 448 of the Civil Code, in relation to Article 546 of the
same Code, which allows full reimbursement of useful
improvements and retention of the premises until
reimbursement is made, applies only to a possessor in good
faith, i.e., one who builds on land with the belief that he is
the owner thereof. It does not apply where ones only interest
is that of a lessee under a rental contract; otherwise, it would
always be in the power of the tenant to improve his

landlord out of his property. 5. Alleged option to buy not


supported by evidence; Promise unenforceable unless option
is in writing Neither the deed of sale over the house nor the
contract of lease contained an option in favor of the Nicolas
spouses to purchase the said lot. The first thing that the
spouses should have done was to reduce the alleged promise
into writing, because under Article 1403 of the Civil Code, an
agreement for the sale of real property or an interest therein
is unenforceable, unless some note or memorandum thereof
be produced. Not having taken any steps in order that the
alleged promise to sell may be enforced, the private
respondents cannot bank Property, 2003 ( 70 ) Haystacks
(Berne Guerrero) on that promise and profess any claim nor
color of title over the lot in question. 6. Option does not
render the Nicolas spouses builders in good faith Even if the
Germinianos indeed promised to sell, it would not make the
spouses possessors or builders in good faith so as to be
covered by the provisions of Article 448 of the Civil Code. The
latter cannot raise the mere expectancy of ownership of the
lot because the alleged promise to sell was not fulfilled nor
its existence even proven. 7. Pecson v. CA does not apply; No
forced co-ownership There is no need to apply by analogy the
provisions of Article 448 on indemnity as was done in Pecson
vs. Court of Appeals, because the situation sought to be
avoided and which would justify the application of that
provision, is not present in the present case. A state of
forced co-ownership would not be created between the
Germinianos and the Nicolas spouses. 8. Lessees governed
by Article 1678 The rights of the lessees are governed by
Article 1678 of the Civil Code which allows reimbursement to
the extent of one-half of the value of the useful
improvements. The right to indemnity under Article 1678 of
the Civil Code, however, arises only if the lessor opts to
appropriate the improvements. Since the Germinianos
refused to exercise that option, the Nicolas spouses cannot
compel them to reimburse the one-half value of the house

and improvements. Neither can they retain the premises until


reimbursement is made. The spouses sole right then is to
remove the improvements without causing any more
impairment upon the property leased than is necessary. [
[G.R. No. 142131. December 11, 2002]

Lacap vs Lee
DECISION
CORONA, J.:

Before us is a petition for review of the decision[1] dated


February 28, 2000 of the Court of Appeals[2] affirming the
decision[3] of the Regional Trial Court (RTC, for brevity) of
Davao City, Branch 11. The said courts affirmed on appeal
the decision dated April 30, 1997 of the Municipal Trial Court
in Cities (MTCC, for brevity) of Davao City in a case[4] for
unlawful detainer filed by respondent Jouvet Ong Lee against
the petitioner spouses Dario and Matilde Lacap.
The facts, as found by the Court of Appeals and the Regional
Trial Court, are as follows:
Before 1981, a certain Victor Facundo mortgaged two parcels
of land and the improvements thereon to Monte de Piedad
Savings Bank (the bank, for brevity). In 1981, herein
petitioner spouses Dario and Matilde Lacap assumed to pay
Facundos mortgage obligation to the bank. Due to their
failure to pay their obligation to the bank, however, the latter
foreclosed on the mortgage. During the auction sale, the
bank emerged as the highest bidder and title passed on to it.

The bank allowed the petitioner spouses to stay in the


premises as lessees paying a monthly rental of P800. The
petitioner spouses introduced improvements thereon
allegedly amounting to some P500,000 after relying on the
banks assurance that the property would be sold back to
them. On May 1, 1996[5], the petitioner spouses
representative went to the bank to pay the monthly rental.
However, the bank refused to accept the rentals inasmuch
as, according to the bank, the property had already been sold
to another person. When the petitioner spouses called the
banks head office, the Vice-President of the Assets Division of
the bank advised them to submit a written offer to the bank
for P1,100,000. The petitioner spouses complied that same
day. But, on May 22, 1996, the bank turned down the
petitioner spouses offer. On June 20, 1996, the petitioner
spouses received a letter demanding that they vacate the
premises because it was already owned by herein
respondent, Jouvet Ong Lee.

The petitioner spouses instituted a civil case against the


respondent for cancellation of sale and damages with an
application for preliminary injunction. This case is now
pending before Branch 13 of the Regional Trial Court (RTC, for
brevity) of Davao City.[6]
Meanwhile, on October 30, 1996, the respondent filed a
complaint for unlawful detainer against the petitioners. After
trial, the Municipal Trial Court of Davao City, Branch 4,
rendered judgment as follows:
WHEREFORE, judgment is hereby rendered in favor of the
plaintiff and against the defendants ordering the latter to:
a) vacate the subject premises;

b) pay P1,500.00 as reasonable compensation for the use of


the said premises commencing the date of this decision until
defendants vacate the same;
c) pay P10,000.00 as and by way of attorneys fees; and
d) cost of suit.
On appeal, the RTC of Davao City, Branch 11, affirmed the
assailed decision of the municipal trial court, with the
modification that respondent should reimburse the petitioner
spouses for the improvements the latter introduced to the
premises. The dispositive portion of the decision reads:
WHEREFORE, premises considered, judgment is hereby
rendered affirming the decision of the court a quo with the
modification that plaintiff should reimburse the defendant for
the improvements the latter introduced on the premises.[8]
The respondent filed a motion for reconsideration praying for
the deletion of the order to reimburse petitioner spouses for
the improvements introduced on the subject premises. On
August 25, 1998, the RTC issued an order granting
respondents motion, to wit:
WHEREFORE, the Motion for Reconsideration of PlaintiffAppellee is hereby granted to leave the premises therein
even if the property may suffer damage. But they shall not
cause more damage than what is necessary. They shall
likewise remove the ornamental improvements introduced
therein.

jurisdiction considering that the issue recovery of the right to


possess was the subject matter of an accion publiciana which
was properly cognizable by the Regional Trial Courts.[10] On
February 28, 2000, the appellate court rendered a decision,
the dispositive portion of which reads:
WHEREFORE, for lack of merit, the instant petition is
DISMISSED and the assailed Decision dated February 20,
1998 and Order dated August 25, 1998 are AFFIRMED.
The appellate court held that the municipal trial court had
jurisdiction over the case inasmuch as the complaint itself
sufficiently alleged that possession was unlawfully withheld
from the respondent who was the registered owner thereof,
and that the petitioner spouses refused to vacate the subject
premises despite demands to vacate the same. In brushing
aside the petitioner spouses argument that respondents
ownership was assailable due to the banks violation of its
promise to first offer the subject property to them, the
appellate court ruled that it could not touch upon said issue
as it was the subject matter of a separate case filed by the
spouses before the RTC of Davao City, Branch 13. Reiterating
the rulings of the courts a quo, the appellate court held that
the petitioner spouses could not be builders in good faith
inasmuch as their payment of rentals to the bank was an
indication that they were lessees. Thus, in the
indemnification for improvements made, Article 1678, not
Article 448, of the Civil Code should govern.

On August 23, 1999, the said court denied the petitioner


spouses motion for reconsideration.

Hence, this petition seeking a resolution on the following


assigned issues:

Petitioner spouses appealed the decision of the RTC to the


Court of Appeals. According to them, the courts a quo
committed serious errors of fact and law in entertaining the
complaint for unlawful detainer despite the lack of

I
WHETHER OR NOT THE COURT OF APPEALS CORRECTLY
RULED ON THE JURISDICTIONAL QUESTION, THAT IS THE

JURISDICTION OF THE DAVAO CITY MUNICIPAL COURT OVER


THE UNLAWFUL DETAINER CASE FILED BY THE RESPONDENT;
AND AS AN ALTERNATIVE TO, BUT ASSOCIATED WITH, THE
ABOVE ISSUE, AND ASSUMING THAT THE DAVAO CITY
MUNICIPAL COURT HAD JURISDICTION,
II
WHETHER OR NOT THE COURT OF APPEALS CORRECTLY
APPLIED ARTICLE 1678 INSTEAD OF ARTICLE 448 OF THE
CIVIL CODE WITH REGARD TO INDEMNITY FOR THE
IMPROVEMENTS INTRODUCED BY THE PETITIONERS ON THE
SUBJECT PROPERTY.[12]
Abandoning their previous position of lack of jurisdiction on
the part of MTC, the petitioner spouses now claim that the
courts a quo erred in oversimplifying the issue in the case at
bar. Since they were questioning the title of the respondent
over the subject property, the case for unlawful detainer was
no longer limited to the question of possession but also
involved the question of ownership. Thus, the courts a quo
should not have evaded ruling on the issue of ownership as a
pre-requisite to the determination and resolution of the issue
of physical possession.
Section 16 of the 1997 Rules of Civil Procedure provides that:
Sec. 16. Resolving defense of ownership.- When the
defendant raises the defense of ownership in his pleadings
and the question of possession cannot be resolved without
deciding the issue of ownership, the issue of ownership shall
be resolved only to determine the issue of possession.

The petitioner spouses are questioning the respondents


ownership by raising as an issue the alleged failure of the
bank to first offer to them the subject property, thereby
making respondents title defective. This, according to the
petitioner spouses, is a defense of ownership that should
have been resolved by the courts a quo.
This Court takes exception to this argument. The defense of
ownership contemplated by the said rule refers to a situation
where the defendants either claim ownership of the subject
property or attributes said ownership to another person other
than the plaintiff. It does not apply where the defendants
merely question the validity of the title of the plaintiff. Thus,
the petitioner spouses must anchor the legality of their
material possession of the property on a claim of title in order
for the court to be able to touch, at least provisionally and
only for purposes of determining possession, on the legality
of the issue of ownership.
In their Reply[13], they do not claim ownership over the
subject premises to support their right to possess the
property. They do not claim having a better right to the said
property by way of transfer of title through one of the modes
of transferring ownership. The alleged violation of their right
of priority or first option to buy the premises is not the
defense of ownership contemplated in Sec. 16 because said
violation, even if true, would only give a cause of action for
damages on the ground of breach of contract but not an
action for recovery of title.
The cases cited by petitioners cannot support their position
as said cases refer to different factual situations. In Oronce v.
Court of Appeals,[14] the defendants maintained ownership
over the property by claiming that the contract of sale with
assumption of mortgage was actually an equitable mortgage.
We ruled therein that the defendant as mortgagor, and not as
vendor, of the property can raise as a defense his claim of

ownership over the subject property. In Refugia v. Court of


Appeals,[15] the defendants claimed title over the subject
property by contending that they, and not the plaintiffs, paid
for the purchase of the said property.

In the instant case, however, the petitioners admit that they


do not own the subject parcels of land. As third persons to
the contract of sale between the bank and the respondent,
they are only questioning the validity of the transfer of title
to respondent. The same cannot qualify as a defense of
ownership as they will not derive title as a consequence but
will, at best, only be given their disputed priority option to
buy the subject premises.

Another reason why the supposed issue of ownership cannot


be ruled upon by the courts a quo is due to the fact that the
same issue is also the subject of a separate pending case for
cancellation of sale filed by the petitioners themselves
against the respondent before the Regional Trial Court Davao
City. In effect, by questioning the ownership of respondent,
the petitioners are raising a defense that serves as the main
cause of action in the complaint for the cancellation of sale
pending before another court. This legal strategy is
prohibited by the rule on the alleged litis pendencia. To ask
the courts a quo to rule on the alleged defense of ownership
is to pre-empt the ruling of the RTC, Branch 13, hearing the
case for cancellation of sale. A party is prohibited from
splitting his cause of action for the reason that it will
unnecessarily clog the court dockets, waste the time and
money of the parties, and perpetrate an abuse of the legal
system by filing cases of the same nature in the hope of
insuring a favorable judgment.

Thus, the ruling of the MTCC that petitioner spouses defense


does not qualify as a defense of ownership is correct.

In the event that their first assigned error is not resolved in


their favor, the petitioner spouses assert that their right to be
indemnified for the improvements they introduced should be
based on Article 448 of the Civil Code which provides that:
Art. 448. The owner of the land on which anything has been
built, sown or planted in good faith, shall have the right to
appropriate as his own the works, sowing or planting, after
payment of the indemnity provided for in Articles 546 and
548, or to oblige the one who built or planted to pay the price
of the land, and the one who sowed, the proper rent.
However, the builder or planter cannot be obliged to buy the
land if its value is considerably more than that of the building
or trees. In such case, he shall pay reasonable rent, if the
owner of the land does not choose to appropriate the building
or trees after proper indemnity. The parties shall agree upon
the terms of the lease and in case of disagreements the
courts shall fix the terms thereof.
Article 546 of the Civil Code provides that builders in good
faith are entitled to reimbursement for necessary and useful
expenses, with right of retention in both cases. The
petitioners insist that they should be treated as builders in
good faith inasmuch as they stepped into the shoes of Victor
Facundo, the former owner-mortgagor, when the latter
assigned to them the obligation to pay the bank the balance
due on the mortgage. Since then, they occupied the subject
property and introduced improvements thereon. They
contend that they were not lessees and paid no rentals
thereon.
We do not think so.

Article 528 of the Civil Code provides that possession in good


faith continues to subsist until facts exist which show that the
possessor is already aware that he wrongfully possesses the
thing. Although, in the beginning, the petitioners were made
to believe that they had a claim of title over the said property
by assuming the mortgage and possessing the subject
property, all this changed when they started paying monthly
rentals to the mortgagee bank after the foreclosure of the
said property. We find this finding of the courts a quo
conclusive on us in this petition for review.[16]
A conclusive presumption arises from the fact that, during
the tenancy relationship, the petitioner spouses admitted the
validity of the title of their landlord. This negated their
previous claim of title.[17] If, indeed, they believed in good
faith they had at least an imperfect title of dominion over the
subject premises, they should have tried to prevent the
foreclosure and objected to the acquisition of title by the
bank. In other words, their supposed belief in good faith of
their right of dominion ended when the bank foreclosed and
acquired title over the subject premises.
Hence, the applicable provision in the instant case is Article
1678 of the Civil Code which provides that:
Art. 1678. If the lessee makes, in good faith, useful
improvements which are suitable to the use for which the
lease is intended, without altering the form or substance of
the property leased, the lessor upon the termination of the
lease shall pay the lessee one-half of the value of the
improvements at that time. Should the lessor refuse to
reimburse said amount, the lessee may remove the
improvements, even though the principal thing may suffer
damage thereby. He shall not, however, cause any more
impairment upon the property leased than is necessary.
With regard to ornamental expenses, the lessee shall not be
entitled to any reimbursement, but he may remove the

ornamental objects, provided no damage is cause to the


principal thing, and the lessor does not choose to retain them
by paying their value at the time the lease is extinguished.
The petitioner spouses are therefore entitled to be paid only
one-half of the value of the useful improvements at the time
of the termination of the lease or to have the said
improvements removed if the respondent refuses to
reimburse them.
WHEREFORE, the petition for review is hereby DENIED. The
decision dated February 28, 2000 of the Court of Appeals is
hereby AFFIRMED. Costs against the petitioners.

Del Campo v. Fernandez-Abesia


[G.R. No. L-49219. April 15, 1988.]
Facts: Spouses Estanislao and Concepcion del Campo and
Bernarda Abesia are co-owners pro indiviso of a lot (Lot 1161
of the Cadastral Survey of Cebu, 45 sq.m. at the corner of F.
Flores and Cavan Streets, Cebu City; TCT 61850) in the
proportion of 2/3 and 1/3 share each, respectively. The
spouses filed an action for partition with the CFI Cebu. The
trial court appointed a commissioner in accordance with the
agreement of the parties. The said commissioner conducted
a survey, prepared a sketch plan and submitted a report to
the trial court on 29 May 1976, recommending that the
property be divided into two lots: Lot 1161-A (30 sq.m.) and
Lot No. 1161-B (15 sq. m.). The houses of the spouses and
Abesia were surveyed and shown on the sketch plan.
Abesias house occupied the portion with an area of 5 sq.m.
of the spouses Lot 1161-A. The parties manifested their
conformity to the report and asked the trial court to finally
settle and adjudicate who among the parties should take
possession of the 5 sq. m. of the land in question. The lower

court held that Article 448 of the New Civil Code does not
apply in the case, and ordered Abesia to demolish at her own
expense part of her house which has encroached Lot 1161-A
and deliver said area to the spouses, with 60 days upon
notice. The parties were ordered to pay the commissioners
fee (P400 in proportional share), and the cost of suit (also in
proportional share). Abesia appealed to the Court of Appeals,
which certified the case to the Supreme Court on account of
the question of law involved, the sole issue is the
applicability of the provisions of Article 448 of the Civil Code
relating to a builder in good faith when the property involved
is owned in common. The Supreme Court modified the
decision appealed from by ordering the spouses to indemnify
Abesia for the Property, 2003 ( 52 ) Haystacks (Berne
Guerrero) value of the said portion of the Abesias house in
accordance with Article 546 of the Civil Code, if the spouses
elect to appropriate the same. Otherwise, Abesia shall pay
the value of the 5 sq. m. of land occupied by their house at
such price as may be agreed upon with the spouses and if its
value exceeds the portion of the house that Abesia built
thereon, Abesia may choose not to buy the land but Abesia
must pay a reasonable rental for the use of the portion of the
spouses land as may be agreed upon between the parties. In
case of disagreement, the rate of rental shall be determined
by the trial court. Otherwise, defendants may remove or
demolish at their own expense the said portion of their
house; Without costs. 1. Article 448 of the Civil Code does not
apply when co-owner built, sown, or planted in good faith, as
he is not a third party; Situation governed by co-ownership
Article 448 of the Civil Code provides that the owner of the
land on which anything has been built, sown, or planted in
good faith, shall have the right to appropriate as his own the
works, sowing or planting, after payment of the indemnity
provided for in articles 546 and 548, or to oblige the one who
built or planted to pay the price of the land, and the one who
sowed, the proper rent. However, the builder or planter

cannot be obliged to buy the land if its value is considerably


more than that of the building or trees. In such case, he shall
pay reasonable rent, if the owner of the land does not choose
to appropriate the building or trees after proper indemnity.
The parties shall agree upon the terms of the lease and in
case of disagreement, the court shall fix the terms thereof.
Article 448 of the Civil Code cannot apply where a co-owner
builds, plants or sows on the land owned in common for then
he did not build, plant or sow upon land that exclusively
belongs to another but of which he is a co-owner. The coowner is not a third person under the circumstances, and the
situation is governed by the rules of co-ownership. 2. Article
448 of the Civil Code applies if co-ownership is terminated by
the partition The provisions of Article 448 of the new Civil
Code should apply when the co-ownership is terminated by
the partition, as in the present case, and when it appears
that the house of Abesia overlaps or occupies a portion of 5
sq. m. of the land pertaining to the spouses which Abesia
obviously built in good faith. Manresa and Navarro Amandi
agree that the said provision of the Civil Code may apply
even when there was co-ownership if good faith has been
established. 3. Options available to the parties Applying
Article 448 of the Civil Code, the spouses have the right to
appropriate said portion of the house of Abesia upon
payment of indemnity to the latter as provided for in Article
546 of the Civil Code. Otherwise, the spouses may oblige
Abesia to pay the price of the land occupied by their house.
However, if the price asked for is considerably much more
than the value of the portion of the Abesias house built
thereon, then the latter cannot be obliged to buy the land.
Abesia shall then pay the reasonable rent to the spouses
upon such terms and conditions that they may agree. In case
of disagreement, the trial court shall fix the terms thereof. Of
course, Abesia may demolish or remove the said portion of
their house, at their own expense, if they so decide.

Ignao v. IAC
[G.R. No. 72876. January 18, 1991.]
Third Division, Fernan (J): 3 concur Facts: A lot situated in
Barrio Tabon, Municipality of Kawit, Cavite, with an area of
534 sq. m. was originally owned by Baltazar Ignao who
married twice. In his first marriage, he had 4 children, namely
Justo (the father of Florencio), Leon, Juan and Isidro. In his
second marriage, Baltazar had also 4 children but the latter
waived their rights over the controverted land in favor of
Justo. Thus, Justo owned 4/8 of the land which was waived by
his half-brothers and sisters plus his 1/8 share or a total of
5/8. Thereafter, Justo acquired the 1/8 share of Leon for
P500.00 which he later sold to his son Florencio for the same
amount. When Justo died, Florencio inherited the 5/8 share of
his father Justo plus his 1/8 share of the land which he bought
or a total of 6/8 (representing 400.5 sq. m.) Juan and Isidro,
on the other hand, had 1/8 share (66.75 sq. m.) each of the
land or a total of 133.5 sq. m. Thus, Florencio and his uncles
Juan and Isidro were co-owners of a parcel of land. Pursuant
to an action for partition filed by Florencio Ignao (Civil Case
N-1681), the then CFI Cavite in a decision dated 6 February
1975 directed the partition of the aforesaid land, alloting
133.5 sq. m. or 2/8 thereof to Juan and Isidro, and giving the
remaining portion with a total area of 266.5 sq. m. to
petitioner Florencio. However, no actual partition was ever
effected. Property, 2003 ( 98 ) Haystacks (Berne Guerrero)
On 17 July 1978, Florencio instituted a complaint for recovery
of possession of real property against Juan and Isidro before
the CFI Cavite (Civil Case 2662). In his complaint, Florencio
alleged that the area occupied by the 2 houses built by Juan
and Isidro exceeded the 133.5 sq. m. previously alloted to
them by the trial court in Civil Case N-1681. An ocular
inspection was conducted by the lower court; which found
that the houses of Juan and Isidro actually encroached upon a
portion of the land belonging to Florencio. Upon agreement of

the parties, the trial court ordered a licensed geodetic


engineer to conduct a survey to determine the exact area
occupied by the houses of Juan and Isidro. The survey
subsequently disclosed that the house of Juan occupied 42
sq. m. while that of Isidro occupied 59 sq. m. of Florencios
land or a total of 101 sq. m. In its decision, the trial court
(thru Judge Luis L. Victor) ruled that although Juan and Isidro
occupied a portion of Florencios property, they should be
considered builders in good faith. The trial court observed
that based on the facts of the case, it would be useless and
unsuitable for Florencio to exercise the first option (of
appropriating part of the house standing on his lot) since this
would render the entire houses of Juan and Isidro worthless.
The trial court then applied the ruling in the similar case of
Grana vs. Court of Appeals, where the Supreme Court had
advanced a more workable solution. Thus, it ordered
Florencio to sell to Juan and Isidro those portions of his land
respectively occupied by the latter at P40.00 per sq. m., and
to execute the necessary deed of conveyance to the Juan and
Isidro; without pronouncement as to costs. Florencio
appealed to the IAC. On 27 August 1985, the Appellate Court
(Second Civil Cases Division), promulgated a decision,
affirming the decision of the trial court. Hence the petition for
review by certiorari. The Supreme Court modified the
decision appealed from. The Court directed Florencio to
exercise his option to either appropriate as his own the
portions of the houses of Juan and Isidro Ignao occupying his
land upon payment of indemnity in accordance with Articles
546 and 548 of the Civil Code, or sell to private respondents
the 101 sq. m. occupied by them at such price as may be
agreed upon; within 30 days from entry of judgment. Should
the value of the land exceed the value of the portions of the
houses that Juan and Isidro have erected thereon, the latter
may choose not to buy the land but they must pay
reasonable rent for the use of the portion of Florencios land
as may be agreed upon by the parties. In case of

disagreement, the rate of rental and other terms of the lease


shall be determined by the trial court. Otherwise, Juan and
Isidro may remove or demolish at their own expense the said
portions of their houses encroaching upon Florencios land;
Without costs. 1. Co-owners hold property in common
dominion, with each an owner of share (abstract and
undetermined until partition is effected Prior to partition, all
the co-owners hold the property in common dominion but at
the same time each is an owner of a share which is abstract
and undetermined until partition is effected. As cited in
Eusebio vs. Intermediate Appellate Court, an undivided
estate is co-ownership by the heirs. As co-owners, the
parties may have unequal shares in the common property,
quantitatively speaking. But in a qualitative sense, each
coowner has the same right as any one of the other coowners. Every co-owner is therefore the owner of the whole,
and over the whole he exercises the right of dominion, but he
is at the same time the owner of a portion which is truly
abstract, because until division is effected such portion is not
concretely determined. 2. Article 448 Article 448 provides
that the owner of the land on which anything has been built,
sown or planted in good faith, shall have the right to
appropriate as his own the works, sowing or planting, after
payment of the indemnity provided for in articles 546 and
548, or to oblige the one who built or planted to pay the price
of the land, and the one who sowed, the proper rent.
However, the builder or planter cannot be obliged to buy the
land if its value is considerably more than that of the building
or trees. In such case, he shall pay reasonable rent, if the
owner of the land does not choose to appropriate the building
or trees after proper indemnity. The parties shall agree upon
the terms of the lease and in case of disagreement, the court
shall fix the terms thereof. Property, 2003 ( 99 ) Haystacks
(Berne Guerrero) 3. Article 448 does not apply to a co-owner;
except co-ownership is terminated In the case of Spouses del
Campo vs. Abesia, it was held that Article 448 of the Civil

Code cannot apply where a co-owner builds, plants or sows


on the land owned in common for then he did not build, plant
or sow upon land that exclusively belongs to another but of
which he is a co-owner. The co-owner is not a third person
under the circumstances, and the situation is governed by
the rules of co-ownership. However, when the co-ownership
is terminated by a partition and it appears that the house of
an erstwhile co-owner has encroached upon a portion
pertaining to another co-owner which was however made in
good faith, then the provisions of Article 448 should apply to
determine the respective rights of the parties. 4. Right to
appropriate works or to oblige builder to pay the price of the
land belongs to the landowner As held in Quemuel vs. Olaes,
it was categorically ruled that the right to appropriate the
works or improvements or to oblige the builder to pay the
price of the land belongs to the landowner. Both the trial
court and the Appellate Court erred when they peremptorily
adopted the workable solution in the case of Grana vs.
Court of Appeals, and ordered the owner of the land,
Florencio, to sell to Juan and Isidro, the part of the land they
intruded upon, thereby depriving Florencio of his right to
choose. Such ruling contravened the explicit provisions of
Article 448 to the effect that the owner of the land shall
have the right to appropriate or to oblige the one who built to
pay the price of the land. The law is clear and unambiguous
when it confers the right of choice upon the landowner and
not upon the builder and the courts. 5. Determination of price
to be paid premature The question on the price to be paid on
the land need not be discussed as this would be premature
inasmuch as Florencio has yet to exercise his option as the
owner of the land.
244 SCRA 477

TECHNOGAS PHILIPPINES vs. CA


121 SCRA 122

G.R. No. 108894 February 10, 1997PANGANIBAN,


FACTS:
The parties in this case are owners of adjoining lots in
Paraaque,Metro Manila. It was discovered in a survey, that a
portion of abuilding of Technogas, which was presumably
constructed by itspredecessor-in-interest, encroached on a
portion of the lot ownedby private respondent Edward Uy.
Upon learning of the encroachment or occupation by its
buildingsand wall of a portion of private respondents land,
the petitioneroffered to buy from defendant that particular
portion of Uys landoccupied by portions of its buildings and
wall with an area of 770square meters, more or less, but the
latter, however, refused theoffer
The parties entered into a private agreement before a
certain Col.Rosales in Malacaang, wherein petitioner agreed
to demolish thewall at the back portion of its land thus giving
to the privaterespondent possession of a portion of his land
previously enclosedby petitioner's wall.
Uy later filed a complaint before the office of Municipal
Engineer of Paraaque, Metro Manila as well as before the

Office of theProvincial Fiscal of Rizal against Technogas in


connection with theencroachment or occupation by plaintiff's
buildings and walls of aportion of its land but said complaint
did not prosper; so Uy dugor caused to be dug a canal along
Technogas wall, a portion of which collapsed in June, 1980,
and led to the filing by thepetitioner of the supplemental
complaint in the above-entitledcase and a separate criminal
complaint for malicious mischief against Uy and his wife
which ultimately resulted into theconviction in court Uy's wife
for the crime of malicious mischief;
ISSUE:
WON the petitioner is builder in good faith.
HELD: YES.
We disagree with Respondent Courts reliance on the cases
of
Tuason & Co.,Inc .vs.Vda.de LumanlanandTuason & Co v
Macalindong
, in ruling that the petitioner "cannot beconsidered in good
faith" because as a land owner, it is "presumedto know the
metes and bounds of his own property, specially if thesame
are reflected in a properly issued certificate of title. One
whoerroneously builds on the adjoining lot should be
considered abuilder in(b)ad (f)aith, there being presumptive
knowledge of theTorrens title, the area, and the extent of the
boundaries." There isnothing in those cases which would
suggest that bad faith isimputable to a registered owner of
land when a part of his buildingencroaches upon a neighbor's
land, simply because he issupposedly presumed to know the
boundaries of his land asdescribed in his certificate of title,
Article 527 of the Civil Code presumes good faith, and since
noproof exists to show that the encroachment over a
narrow,needle-shaped portion of private respondent's land

was done inbad faith by the builder of the encroaching


structures, the lattershould be presumed to have built them
in good faith. It ispresumed that possession continues to be
enjoyed in the samecharacter in which it was acquired, until
the contrary is proved.
Good faith consists in the belief of the builder that the land
he isbuilding on is his, and his ignorance of any defect or flaw
in histitle. Hence, such good faith, by law, passed on to
Pariz'ssuccessor, petitioner in this case. The good faith
ceases from themoment defects in the title are made known
to the possessor, byextraneous evidence or by suit for
recovery of the property by thetrue owner.
Consequently, the builder, if sued by the aggrieved
landowner forrecovery of possession, could have invoked the
provisions of Art.448 of the Civil Code, which reads:
The owner of the land on which anything has been built,sown
or planted in good faith, shall have the right toappropriate as
his own the works, sowing or planting,after payment of the
indemnity provided for in articles546 and 548, or to oblige
the one who built or planted to pay the price of the land, and
the one who sowed, the proper rent. However, the builder or
planter cannot beobliged to buy the land if its value is
considerably morethan that of the building or trees. In such
case, he shall pay reasonable rent, if the owner of the land
does not choose to appropriate the building or trees after
property
indemnity. The parties shall agree upon the terms of thelease
and in case of disagreement, the court shall fix theterms
thereof.
The obvious benefit to the builder under this article is
that,instead of being outrightly ejected from the land, he can
compelthe landowner to make a choice between the two
options: (1) toappropriate the building by paying the

indemnity required by law,or (2) sell the land to the builder.


The landowner cannot refuse toexercise either option and
compel instead the owner of thebuilding to remove it from
the land
In view of the good faith of bothpetitioner and
privaterespondent, their rights and obligations are to be
governed by Art.448. Hence, his options are limited to: (1)
appropriating theencroaching portion of petitioner's building
after payment of proper indemnity, or (2) obliging the latter
to buy the lot occupiedby the structure. He cannot exercise a
remedy of his own liking.

Quemuel v. Olaes
[G.R. No. L-11084. April 29, 1961.] En Banc, Paredes (J): 9
concur
Facts: Angel S. Olaes and his wife, Juliana Prudente,
registered owners of lot 1095 of the San Francisco de
Malabon Estate, located in Rosario, Cavite, sought the
recovery of the possession of the said lot and rentals
therefore in Civil Case 5442 CFI Cavite, from Alejandro
Quemuel and his wife Ruperta Solis, who claimed to be in
possession under the tolerance of the former. On 16 March
1954, the trial court ordered Quemuels to return the
possession of lot 1095 to the Olaes spouses and to pay the
latter P20 a month from January 1954, until they shall have
vacated the premises. The Quemuels did not appeal from
said decision which became final on 22 April 1954.
Thereafter, the Olaes spouses sought the execution of the
decision. To forestall ejectment, the Quemuels filed on 1 July
1954 the complaint (Civil Case 5518, CFI Cavite). The
Quemuels seek to reduce the monthly rental of P20 fixed in
Civil Case 5442, and to compel the Olaes spouses to sell to
them the portion of the lot 1095 where their house is

erected. The Olaes spouses filed a motion to dismiss on 9 July


1954, alleging lack of cause of action, res adjudicata;
prescription, and the cause of action, if any, is barred by the
Quemuels failure to set it up as a counter-claim in Civil Case
5442. On 17 September 1954, the trial court dismissed the
complaint, without pronouncement as to costs. An appeal
was taken by the Quemuels to the Court of Appeals (CA-GR
14837-R) which, by the agreement of the parties, certified
the case to the Court. The ex parte petition filed by the
Quemuels in the Supreme Court on 9 August 1956, asking
that a writ of preliminary injunction be issued to the
Provincial Sheriff of Cavite and the Olaes, enjoining them
from demolishing the house of the Quemuels until there is a
final decision in said Case 14837, was denied on 14 August
1956. The Supreme Court affirmed the order appealed from,
with costs against the Quemuels. 1. The Complaint states no
cause of action; If the rent imposed by court are found
excessive, the Quemuels are free to vacate the property A
cause of action presupposes a right of the plaintiff and a
violation of such right by the defendant. According to the
complaint itself, the rental of P20 monthly and the order to
vacate, were provided in a prior judgment (Civil Case 5442)
which is final and its validity is not assailed. There being no
law that fixes the rental of the same land at 7 1/2% of its
alleged market value, the plaintiffs have no right thereto, or a
right which could be violated. The defendants are not
compelling the plaintiffs to rent the property but wanted
them to vacate the premises (Civil Case 5442). If the rental
determined by the trial court were excessive, the plaintiffs
are free to vacate the property. For plaintiffs to insist on
possessing the property and fixing the rentals themselves,
would have no legal sanction at all. 2. Builder in good faith:
Right to appropriate works or improvements belong to owner
of the land, Property, 2003 ( 162 ) Haystacks (Berne
Guerrero) builders right to reimbursement; Builder cannot
compel owner of the land to sell land Under Article 448, the

right to appropriate the works or improvements or to oblige


the one who built or planted to pay the price of the land
belongs to the owner of the land. The only right given to the
builder in good faith is the right to reimbursement for the
improvements; the builder, cannot compel the owner of the
land to sell such land to the former. This is assuming that the
plaintiffs are builders in good faith. 3. Quemuels not builders
in good faith; Builder builds under claim of title The
Quemuels are not builders in good faith. Article 448 of the
new Civil Code, (equivalent to Article 361 of the old Civil
Code) is intended to apply only to a case where one builds, or
sows, or plants on land in which he believes himself to have a
claim of title and not to lands wherein ones only interest is
that of tenant, under a rental contract, which is the present
case (Alburo vs. Villanueva, 7 Phil., 277). The tenant cannot
be said to be a builder in good faith as he has no pretension
to be owner (Rivera vs. Thailand, 48 Phil., 396; see also 3
Manresa 4th Ed. pp. 215-216). 4. Quemuels are lessees From
the pleadings and the documentary evidence submitted, it is
indisputable that the land in question originally belonged to
the government as part of the Friar Lands Estate and the title
thereto was in the name of the government until it was
purchased by Agapita Solis who applied, thru the Bureau of
Lands, to purchase the land by installments. The
corresponding Sale Certificate 531, effective 1 July 1909 was
executed. In Olaes spouses complaint, they alleged that they
are the owners of lot 1095 and that the Quemuels have been
occupying southeastern half portion thereof, without any
right thereto, except the tolerance of the Olaes spouses. The
Quemuels were not unaware of the flaw in their title, if any,
and that their true relation with the Olaes spouses was that
of tenant and landlord, and, that their rights are governed by
Article 1573 in relation to article 487 of the old Civil Code.
Article 1573 provides that a lessee shall have with respect
to useful and voluntary improvements, the same right which
are granted the usufructuaries. Article 487 provides that

the usufructuary may make on the property held in usufruct


any improvements, useful or recreative, which he may deem
proper, provided he does not change its form or substance,
but he shall have no right to be indemnified therefor. He may,
however, remove such improvements, should it be possible
to do so without injury to the property. 4. Lessee cannot
compel lessor to pay for the improvements or to sell the land;
Right to remove improvements if it can be done without
damage to the land From Articles 487 and 1573, it can clearly
be inferred that the Quemuels cannot even compel the Olaes
spouses to pay for the improvements the former made in the
property or to sell the latters land. The Quemuels only right,
is to remove the improvements, if it is possible to do so
without damage to the land. 5. Identity of case 5442 and
5518; Assuming otherwise, covered by Res judicata In Case
5442, the matter of the rental was in issue, and the same
was considered and decided by the trial court, which ordered
the Quemuels therein to pay a reasonable compensation of
P20 a month beginning with January, 1954, until they shall
have left the premises. In the present case (5518), the
parties are the identical parties in Civil Case 5442, the same
lot 1095 is the subject matter of both cases; the same issue,
namely, the amount of the rental is involved. Even assuming
that Quemuels have a cause of action, the doctrine of res
judicata already operates against them. 6. Lot purchased by
Agapita Solis; Assuming otherwise, covered by Prescription
Lot 1095 was purchased by Agapita Solis from the
Government on 1 July 1909. After full payment of the
purchase price, TCT 10771 covering said lot was issued to
said Agapita Solis on 8 June 1933. Assuming that the
Quemuels or their alleged predecessor-in-interest, had a
cause of action for claiming the ownership of portions of said
lot, such cause of action accrued at the latest on 8 June
1933. The statute of limitations provide that the Quemuels or
their predecessor had 10 years from said date, within which
to file the corresponding action; which cannot be had as the

Quemuels filed the complaint after more than 21 years or on


Property, 2003 ( 163 ) Haystacks (Berne Guerrero) 1 July
1954. 7. Failure to set up a counterclaim in Civil Case 5442
Whether the cause of action is for recovery of ownership or
for an alleged right to purchase the property, or for
reimbursement for some improvements, the Quemuels
should have set it up as a counterclaim in Civil Case 5442,
because same was necessarily connected with, or arose out
of the transactions involved in Civil Case 5442 (Section 6,
Rule 10, Rules of Court). 8. De Jesus v. Belarmino does not
apply In De Jesus, et al. vs. Belarmino, et al. (GR L-6665, 30
June 1954; OG July 1954, p. 3064), it was held that where
the complaint was dismissed not because of any evidence
presented by the parties, or as a result of a trial on the
merits, but merely on a motion to dismiss filed by the
defendants, the sufficiency of the motion should be tested on
the strength of the allegation of facts contained in the
complaint and no other, which has been interpreted to apply
to cases where the motion to dismiss is based solely on the
ground of lack of cause of action. Considering the fact that
(1) In the present case, documentary evidence and the
records of the Civil Case 5442 were presented and
considered by the trial court; and (2) in the De Jesus case,
the only ground for dismissal was the lack of cause of action,
while in the present case, aside from said ground, plaintiffs
alleged other grounds, the said ruling finds no application.
San Diego v. Montesa
[G.R. No. L-17985. September 29, 1962.] En Banc, Reyes JBL
(J): 8 concur
Facts: After trial in Civil Case 770 of the CFI Bulacan, on
complaint of Jose, Maria, and Urbano, all surnamed de la
Cruz, to recover a parcel of land and damages from Gil San
Diego and Rufina San Diego, the Court rendered a decision
declaring the deed of sale null and void, ordering the San

Diegos to vacate the land upon payment of the sum P3,500


by the de la Cruzes within 30 days after the decision
becomes final, dismissing the counterclaim of the de la
Cruzes, without pronouncement as to costs. The court found
that the disputed portion of a parcel of land belonged to the
de la Cruzes through hereditary succession; that the San
Diegos built a house on the land in good faith, having
acquired the land from Catalina Anastacio, mother of the de
la Cruzes, by purchase for P1,000.00. During the
proceedings, the San Diegos filed a third-party complaint
against said vendor. The vendor (mother of the de la Cruzes)
subsequently died; hence, the de la Cruzes, who were the
plaintiffs, became at the same time third-party defendants in
substitution of their deceased mother. The court voided the
sale on the ground that the vendor had no right to the land,
but upheld the defense of the San Diegos as builders in good
faith. On appeal by the de la Cruzes, the Court of Appeals
affirmed in toto the lower courts decision, and the same,
thereafter, became final and executory. Over 2 years later,
the San Diegos, who were in possession of the parcel of land
in litigation, moved to execute paragraph (b) of the
dispositive portion of the decision in order to collect the sum
of P3,500.00 and thereafter to vacate the premises. The
motion was denied by the court, and a motion for
reconsideration was likewise of no avail. Hence, the instant
petition for mandamus was filed to compel the judge to issue
the writ applied for. The Supreme Court granted the writ
prayed for, and ordered the CFI Bulacan to issue the writ of
execution in favor of San Diegos. Costs against de la Cruz 1.
Judgment based on right of retention due to possessors in
good faith; No rental required during period of retention The
judgment affirmed by the Court of Appeals, and now final,
explicitly ordains the payment by the de la Cruzes of the
amount of P3,500.00 within 30 days after this decision
becomes final to the San Diegos. If it also orders the San
Diegos to vacate only upon such payment, it did so in

recognition of the right of retention granted to possessors in


good faith by Article 546 of the Civil Code of the Philippines.
This provision is expressly made applicable to builders in
good faith (Article 448). The right of retention thus granted is
merely a security for the enforcement of the possessors
right to indemnity for the improvements made by him. As a
result, the possessor in good faith, in retaining the land and
its improvements pending reimbursement of his useful
expenditure, is not bound to pay any rental during the period
of retention; otherwise, the value of his security would be
impaired (cf. Tufexis vs. Chunaco (C.A.) 36 O.G. 2455). 2.
Options of the landowner; Decision limited to appropriation of
the improvement which the landowner did not object to;
Decision final and cannot be altered Normally the landowner
has the option to either appropriate the improvement or to
sell the land to the possessor. This option is no longer open
to the landowners herein because the decision in the former
suit limits them to the first alternative by requiring the San
Diegos to vacate the land (and surrender the improvements)
upon payment of P3,500.00. Evidently, the CFI and the CA
opined that the de la Cruzes suit Property, 2003 ( 181 )
Haystacks (Berne Guerrero) to recover the property was an
exercise of their right to choose to appropriate the
improvements and pay the indemnity fixed by law. The de La
Cruzes acquiesced in this view, since they did not ask for the
modification of the judgment and allowed it to become final.
Consequently, they can no longer insist on selecting another
alternative; nor can they be heard now to urge that the value
of the indemnity, set at P3,500.00, is exorbitant, for the same
reason that the judgment fixing that amount is no longer
subject to alteration. 3. Courts duty to execute a final
judgment; Writ of Mandamus The judgment ordering
payment to the San Diegos of P3,500.00, by way of
indemnity, having become final, and the 30 days for its
payment having elapsed, the CFI has the ministerial duty to
order its execution (Zulueta vs. Paredes, 62 Phil. 5;

Buenaventura vs. Garcia, 78 Phil. 759; Amor vs. Jugo, 17 Phil.


703; Viquiera vs. Baraa, 78 Phil. 456). That duty is
compellable by mandamus; and the execution is leviable on
any property of the de la Cruzes, including the land now in
question and its improvements.
PNB V. DE JESUS
411 SCRA 557
FACTS:It would appear that on 10 June 1995,respondent filed
a complaint against petitioner beforethe Regional Trial Court
of Occidental Mindoro for recovery of ownership and
possession, withdamages, over the questioned property. In
hiscomplaint, respondent stated that he had acquired aparcel
of land situated in Mamburao, OccidentalMindoro, with an
area of 1,144 square meterscovered by TCT No. T-17197, and
that on 26 March1993, he had caused a verification survey of
theproperty and discovered that the northern portion of the
lot was being encroached upon by a building of petitioner to
the extent of 124 square meters.Despite two letters of
demand sent by respondent,petitioner failed and refused to
vacate the area.Petitioner, in its answer, asserted that whenit
acquired the lot and the building sometime in 1981from then
Mayor Bienvenido Ignacio, theencroachment already was in
existence and toremedy the situation, Mayor Ignacio offered
to sellthe area in question (which then also belonged
toIgnacio) to petitioner at P100.00 per square meter which
offer the latter claimed to have accepted. Thesale, however,
did not materialize when, without theknowledge and consent
of petitioner, Mayor Ignaciolater mortgaged the lot to the
Development Bank of the Philippines. He also contends that
he is a builder in good faith.
ISSUE:Whether or not being a builder in good faith
mattersunder article 448.

HELD:Article 448, of the Civil Code refers to apiece of land


whose ownership is claimed by two or more parties, one of
whom has built some works (or sown or planted something)
and not to a case wherethe owner of the land is the builder,
sower, or planter who then later loses ownership of the land
by sale or otherwise for, elsewise stated, where the true
owner himself is the builder of works on his own land,
theissue of good faith or bad faith is entirely irrelevant.
304 SCRA 37

Santos v. Mojica
[G.R. No. L-25450. January 31, 1969.] En Banc, Capistrano (J):
10 concur, 1 took no part
Facts: On 19 March 1959, Teodorico, Carmen, Antero, Vidal,
Catalina, Melanio, Manuel, Felicidad, Aurelio, Pacita and
Eleuteria, all surnamed Allanigue (being brothers and
sisters), brought an action (Civil Case 217- R) before the CFI
Rizal against their sister, Lorenza Allanigue, her husband,
Simeon Santos, Maria San Agustin and Felicidad San Agustin,
for partition of a 360-sq. m. lot situated at San Dionisio,
Paraaque, Rizal, and for the annulment of certain
conveyances involving the same. Defendants having been
declared in default, the trial court, after hearing the plaintiffs
evidence, rendered judgment ordering the partition of the lot
among the 11 plaintiffs and the defendant Lorenza Allanigue.
In a subsequent order the court set off Lorenza Allanigues
share against the amount that she had failed to pay as rents
to the plaintiffs as directed in the decision. A writ of
execution was issued on the judgment ordering the
defendants to vacate the lot and deliver its possession to the
plaintiffs. Leonardo Santos, not a party defendant but a son
of defendants Simeon Santos and Lorenza Allanigue, owned a
house standing on the lot. He filed with the sheriff a thirdparty claim, and with the court, a motion to recall the writ of
execution insofar as his house was concerned. The motion
was denied. On 15 March 1962, the defendants and movant
Leonardo Santos having failed to remove their houses from
the lot within the period given them, the court ordered the
sheriff to demolish said houses. On 2 April 1962, Leonardo
Santos and the defendants in the case, as petitioners, filed in
the Supreme Court a petition for certiorari and prohibition
(GR L-19618), against Judge Angel H. Mojica, the Provincial
Sheriff of Rizal and the plaintiffs in the case, as respondents.
In its decision of 28 February 1964, the Supreme Court
denied the petition after finding that Leonardo Santos did not

follow the procedure sanctioned by law in vindicating his


alleged ownership, i.e., he should have filed an ordinary civil
action to vindicate his alleged ownership of the house and
the portion of land on which it was built. After the said
decision of the Supreme Court had become final, the lower
court (Judge Angel H. Mojica), on motion of the plaintiffs in
the same Civil Case 217-R, ordered the demolition of the
defendants houses. The defendants having voluntarily
removed their houses, the only house that remained standing
on the lot was that belonging to Leonardo Santos.
Subsequently, the Judge, on motion of the plaintiffs, issued
an order dated 9 December 1965, directing the sheriff to
demolish the house of Leonardo Santos. Hence, the present
petition for certiorari and prohibition in the Supreme Court.
The Supreme Court denied the petition, with costs against
the petitioner Leonardo Santos. 1. Successor-in-interest
bound by the judgment in Civil Case 217-R Property, 2003
( 182 ) Haystacks (Berne Guerrero) Leonardo Santos is bound
by the judgment in Civil Case No. 217-R because he is a
successor-in-interest of his parents, Simeon Santos and
Lorenza Allanigue, defendants in Civil Case 217- R, and his
right, if any, is claimed under them. The fact that the sale to
Leonardo Santos from his parents was registered, is of no
moment because, he is bound by the judgment against them.
2. Improvement of the house made after predecessor-ininterest were summoned; Santos a builder in bad faith, no
right of indemnity Leonardo Santos house having been built
and reconstructed (after March 1962) into a bigger one after
his predecessors-in-interest, his parents, had been
summoned in 1959 in Civil Case 217-R, he must be deemed a
builder in bad faith. As builder in bad faith he lost the
improvement made by him consisting of the reconstructed

house to the owners of the land without right to indemnity,


pursuant to Article 449 of the Civil Code (He who builds,
plants or sows in bad faith on the land of another, loses what
is built, planted or sown without right to indemnity.) 3.
Options of the landowner in good faith The owners of the
land became owners of the improvement consisting of the
house built in bad faith if they chose to appropriate the
accession. (Article 445 and 449, Civil Code.) However, said
owners could choose instead the demolition of the
improvement or building at the expense of the builder,
pursuant to Article 450 of the Civil Code, which, in part,
provides The owner of the land on which anything has been
built, planted or sown in bad faith may demand the
demolition of the work, or that the planting or sowing be
removed, in order to replace things in their former condition
at the expense of the person who built, planted or sowed. In
the present case, the Allanigue brothers and sisters chose to
have the house or improvement built by Leonardo Santos
demolished pursuant to their motion for demolition. 4. Res
Judicata; Identity of the case The present petition is barred by
the prior judgment of the Court in GR L-19618. Petitioner
Leonardo Santos, was one of the petitioners in that case
against the same official and private respondents in the
instant petition; having an identity of subject matter (the
portion of the lot and the house standing on said portion
alleged by petitioner to belong to him), and identity of cause
of action (the order of the Judge for the removal or demolition
of the houses standing on the lot). The prior judgment, based
on merits, had become final. The judgment thus in GR L19618 is res judicata in the instant case on the question of
the validity of the order of demolition of 9 December 1965.

Vous aimerez peut-être aussi